Cardiology UWorld/Pastest

The use of 5PDE inhibitors is contraindicated in patients using _________________ for the treatment of angina, as the combination of these two drugs may cause life-threatening hypotension.

Nitrates (ex. nitroglycerin)

IV drug users usually manifest with _______________ valve abnormalities

Tricuspid valvue.
> When the organism enters the venous circulation, on its way to the heart, the first value it encounters is the tricuspid valve.

The FROM JANE mnemonic describes the symptoms of?

Bacterial endocarditis
FROM JANE
F - Fever
R - Roth's spots
O - Osler nodes
M- Murmur
J - Laneway lesions
A - Anemia
N - Nail bed hemorrhages
E - Emboli.
The most common infectious organism is IV drug supers is S. aureus.
FYI - Crackles upon lung ausculta

Tetralogy of Fallot presents with pulmonary stenosis, VSD, overriding aorta and right ventricular hypertrophy. Abnormal migration of the __________________, is responsible for malalignment (anterosuperior displacement) of the infundibular septum

Neural crest cells

A man presents with fever, recent upper respiratory infection, pleuritic test pain, and diffuse ST-segment elevation with PR-segment depression on ECG consistent with a diagnosis of _________________________

Acute pericarditis.
> Pleuritic chest pain is aggravated by deep inspiration and movements that increase pericardial stretch.
> Acute pericarditis is most often due to a viral infection

An apical diastolic rumble with opening snap is heard in __________________

mitral stenosis

A high-pitched apical pan systolic murmur radiating to the axilla is characteristic of ____________________

mitral regurgitation

A continuous systolic and diastolic murmur obscuring the S2 sound and radiating to the back is heard in ______________________

coarctation of the aorta

A patient has both diabetes and hypertension. These type of patients require a specific combination of t antihypertensive medication that not only lowers the blood pressure but also is protective of the kidneys. Diabetes mellitus can have detrimental effe

ARBs (angiotensin II receptor blockers)
And
ACEi

In ____________________, patients typically present it hypertension in the upper extremities and relative hypertension in the lower extremities with occasional leg claudication symptoms.

Aortic coarctation - characterized by narrowing of the aortic lumen typically in a post ducal location.
> This is because blood shunts preferentially through the arch vessels flow is reduced through the descending aorta.

What are the symptoms of renal artery stenosis?

Renal artery stenosis - narrowing of the renal artery
- Secondary hypertension
> This causes decreased blood flow to the kidneys, which can be in turn cause fluid retention and hypertension.
However, a BRUIT is typically heard on auscultation of the abdom

In ________________. a constant, machine-like murmur may be detected on cardiac auscultation

Patent ductus arteriosus .
> Patients with a patent ductus have a sleet-to-right shunt that can result in left ventricular dilatation and failure if untreated.

In _______________ there is left-to-right shunt and can be detected by a fixed, widely split S2 on cardiac auscultation

Atrial septal defect

In the presence of secondary hypertension, you should always first rule out _____________ and ________________

Nicotine AND oral contraceptives.

What is most likely the diagnosis? A 4 y/o boy from China with restlessness, chest pain, and abdominal pain. Approximately 8 months ago, he had a suspected viral illness with fever, cervical lymphadenopathy, erythematous oropharynx, and a desquamating ras

Kawasaki disease
Patients with Kawasaki disease may develop coronary artery thrombosis, which can lead to myocardial infarction
The symptoms of Kawasaki disease may be remembered with the mnemonic "CRASH and burn":
- C: Conjunctivitis
- R: Rash (polymorph

Since the _____________ nerve runs along the carotid artery it may be damaged during carotid endarterectomy, leading to difficult to control severe hypertension.
Hoarseness and dysphagia are also supportive of vagus nerve injury, in particular, the recurr

vagus nerve

____________________ usually follows infection with enterohemorrhagic E. coli. In addition the presence of renal failure and a microangiopathichaemolytic anemia.

Hemolytic uremic syndrome

Idiopathic thrombocytopenia purpura is not associated with a ______________ platelet count and the purpic rash is not ____________________

Normal; palpable

_____________________ is a small-vessel vasculitis characterized by palpable purpura - usually located on the buttock area and lower extremities. Additional clinical manifestations include arthralgia, diarrhea, abdominal pain and glomerulonephritis.

Hence-Schonlein purpura
Usually seen in children ages of 4 to 7 years old.

The following symptoms of characterized by which disorder? Diffuse chest pain that alleviated upon leaning forward. Temperature of 37.9 degrees C, blood pressure of 140/84 mmHg, pulse of 76/min. Cardiac auscultation reveals a faction rub. Electrocardiogra

Pericarditis - inflammation of the pericardium
Most likely due to, Post-viral complication
> The patient reported symptoms of a recent viral infection (sluggishness and sore throat), and then developed pericardial symptoms acutely.
A friction rub is often

_________________ syndrome refers to post-myocardial infarction pericarditis. There are ST elevation but it is NOT diffuse (diffuse represents transmural infarction)

Dressler
Dressler syndrome - pericarditis that arises 6-8 weeks after infarction due to an autoimmune phenomenon where you began to produce antibodies against your own pericardium

Pericarditis can be cased by ____________, but we would expect other symptoms such as rash, myalgaia, or joint pain. A positive antinuclear antibodies test is also likely.

Lupus (SLE)

_____________________ such as simvastatin, lovastatin, and atorvastatin are used in patients with high low-density lipoprotein (LDL) levels. Inhibition of HMG CoA reductase decrease cholesterol formation via the cholesterol pathway.
Common adverse effects

HMG CoA reductase inhibitors
> The creatinine phophokinase (CPK) level should be checked in patients with statin-induced myopathy and if it is more than 10 times the normal level the statin should be stopped.
> Liver function tests should also be checked

______________________ is a bile acid resin. It binds with bile salt and prevents reabsorption of cholesterol from the GI that. Common adverse effects include GI upset such as constiptation, diarrhea and bloating, but not muscle ache. It also decreases re

Cholestryamine

____________________ prevents cholesterol absorption from the GI tract. It is not used as a single therapy, rather in conjunction with a statin.

Ezetimbie.

_________________ decrease VLDL, LDL, and triglyceride and slightly increase HDL levels. They are most commonly used in patients with very high triglyceride levels.
Adverse effects include gall stone formation, rash, myositis.
They usually cause muscle da

Vibrates.

__________________ is used in patients with hyperlipidemia. It inhibits very low-density lipoprotein (VLDL), decreases LDL and triglyceride, and increases high-density lipoprotein (HDL) levels.
Common adverse effects include flushing and itching, but not

Nicotinic acid

A 4-mouth old infant is brought to the physician for a routine examination. Physical examination shows epicanthic folds and a palmar simian crease. Cytogenetic texting shows an autosomal trisomy. Which cardiac anomalies is most common associated with this

Complete atrioventricular septal defect
> The description with the presence of epicanthic folds and a simian crease should alert to the diagnosis of Down's syndrome or trisomy 21. Complete atrioventricular septal defect is the most common cardiac abnormal

Case: A 75 yo man is brought to the ED because of a 2-hour history of crushing, substernal pain that radiates to the left jaw. He is administered morphine, oxygen, sublingual nitroglycerin, and metoprolol. One hour later, he has ventricular fibrillation,

No change
This clinical vignette is consistent with acute myocardial infarction (MI). Medical treatment for MI includes oxygen, beta-blocker, nitroglycerin, aspirin, and morphine for pain. If percutaneous coronary intervention (PCI) is available, gold sta

Case: A 64 y/o is brought to he ED after complaining of crushing best pain and shortness of breath. ECG reveals ST-segment elevations in leads I, aVL, and V5, V6. CK-MB levels are also elevated. She is treated according to protocol for her condition. Howe

Ventricular fibrillation
This patient pretend to the ED experiencing a MI. In patients with MI, the MOST COMMON cause of sudden death is arrhythmia, particularly ventricular fibrillation.
Complications
> 0-24 hrs: Arrhythmia, cariogenic shock, heart failu

Case: A 42 y/o man comes to the physician because of High blood pressure. Physical examination shows yellow flat plaques of the lower eyelids. He says it has been present for many years. His father died of a MI at 50 years old. He feels well and exercises

Tendon xanthomata
> The appearance is of yellow flat plaques over the lower eyelids. They represent areas of lipid-containing macrophages. The diagnosis is xanthelasma.
> It is possible given the premature death of his father that he has familial hypercho

What kinds of diseases do you see an enlarged liver?

An enlarged liver or hepatomegaly is present in conditions like
1) Hepatitis (infectious, drug-induce, alcoholic, or nonalcoholic)
2) Storage disorders (glycogen storage diseases or hemochromatosis)
3) Impaired hepatic venous outflow (Budd-chairir or righ

What condition would you see
speckled iris
(Brushfield spots)

A speckeld iris (brush field spots) is due to small white or greyish/brown spots on the periphery of the iris due to aggregation of connective tissue These are normal in children, but are also a feature of Down syndrome.

Case: A 64 yo man is brought to the ED because of pleuritic chest pain over the past 3 hours. The pain began while lying in bed. He has a history of diabetes mellitus and a MI 4 weeks prior, for which he underwent coronary artery bypass surgery. The surge

Dressler syndrome
> Dressler syndrome is a complication of transmural myocardial infarction thought to arise from formation of autoantibodies against cardiac antigens released from necrotic myocytes.
> Symptoms include mild fever and pleuritic chest pain

Case: A 16 y/o boy with beta thalassemia requiring regular blood transfusions is diagnosed with heart filature and new onset diabetes mellitus. He is placed on the waiting list for a heat transplant.
What is the most likely cause of heart failure in this

Iron overload cardiomyopathy
> Frequent blood transfusion result in the
accumulation and deposition of iron
in various organs including the liver, heart, and pancreas. Iron overload cardiomyopathy is a term coined to a secondary cause of cardiomyopathy re

An _______________________ is due to mismatch between major histocompatibility antigen on blood cells, such as the ABO system. It causes severe intravascular hemolysis, disseminated intravascular coagulation, renal filature and shock and has a high mortal

Acute hemolytic transfusion reaction

Case: A 3 y/o boy is brought to the ED because of high fever, cervical lymph node enlargement, conjunctival congestion, redness of lips and palms, and desquamation of fingertips. Physical examination shows erythema of the oral cavity. Lab show hemoglobin

Kawasaki diseases (mucocutaneous lymph node syndrome)
Kawasaki diseases, also known as mucocutaneous lymph node syndrome, is an acute, fertile, multistyem disease of children. Most cases occur prior to the age of 5. A coronary arteritis is an important ha

A prolonged PR interval (defined as >200 ms) with 1:1 conduction ratio of P waves to QRS complies is characteristic of _______________________

1st degree heart block.
> First Degree Block - A conduction block that occurs at the AV node (or the bundle of His, which is immediately distal)
> Every atrial impulse does eventually get conducted
> The only thing that you need to diagnose a First Degree

2nd degree heart block, _________________________ is characterized by progressive lengthening of the PR interval until a QRS complex is dropped. It is usually symptomatic, of no clinical significance and therefore does not require treatment.

Mobritz type 1 or Wenkebach phenomenon

2nd degree heart block, _______________________ is characterized by consistent unchanging PR intervals (usually normal in duration but can be prolonged) followed by the block of one or more P waves that fail to conduct to the ventricles.

Moritz type 2
> Moritz type 2 is a high degree AV block with the potential for progressing to complete heart block

___________________________ is identified by P waves and QRS complexes that are independent of each other

Complete heart block (or 3rd degree heart block)

A normal PR interval is _____________ ms

< 200

Case: A 1-day-old term new born of a diabetic mother is evaluated for blue discoloration of the skin, clubbing of the fingers, poor feeding, and severe shortness of breath. Physical examination shows lethargy and cyanosis. He is started on supplemental ox

Failure of migration of neural crest and endocardial cells
> The differential diagnosis of neonatal cyanosis and respiratory distress is broad, including aspiration of meconium or amniotic fluid, congenital heart disease, diaphragmatic herina, bronchopulm

Atrial septal defects (ASD) are associated with a _____________ heart sound on physical exam.

Fixed-split S2
> Similar to a VSD, ASD is typically not associated with cyanosis, unless left untreated for a significant period of time, which may lead to Eisenmenger's syndrome that results in cyanosis. However, ASD would not present with cyanosis on th

Failure of complete formation of the membranous inter ventricular septum results in a ventricular septal defect (VSD), which is typically associated with a _________________heart murmur.

holosystolic
> VSD is typically not associated with cyanosis, especially in the first days of life.
Large VSDs, if left untreated may result in Eisenmenger's sydrnome, which results in a right to left intracardiac shunt and, ultimately, cyanosis.

Case: A 42 yo man with allergic rhinitis and asthma is treated with inhaled bronchodilators, cromolyn sodium, and anti-inflammatory agent montelukast. He develops mild fever, malaise, weight loss, night sweats, multiple joint pain, tingling sensation in b

Churg-Strauss syndrome
>This patient is suffering from eosinophilic granulomatosis with polyangiitis Churg-Strauss syndrome, which is characterized by a triad of:
1) Late-onset asthma/allergic rhinitis
2) Systemic vasculitis (presenting with constitutiona

Peripheral eosinophilia is a component of ___________________________ syndrome

Churg-Strauss

The clinical presentation of classic polyarteritis nods is similar to that of Churg-Strauss syndrome, except that it is not associated with _______________ or ________________. It typically does NOT involve the lung. Glomerulonephritis and gastrointestina

Asthma; esoinophilia

Granulomatosis with polyangiitis (GPA) is a form of non-caveating granulomatous, necrotizing vasculitis. The classic triad of organ involvement consist of:
1) ______________
2) ______________
3) ______________

1) Upper respiratory tract and sinuses (leading to chronic sinusitis, rhinitis, nosebleed, septal perforation and saddle nose deformity)
2) Lungs (leading to hemoptysis and pulmonary cavitary lesions)
3) Kidneys (leading to necrotizing glomerulonephritis

______________________ is a small-vessel vasculitis. It is similar to GPA, except that it is not granulomatous in nature and does not involve upper respiratory tract. p-ANCA is positive this as well.

Microscopic polyangiitis

Case: A newborn, with Apgar scores of 8 and 9 at 1 and 5 minutes respectively, is examined for suspected congenital anomaly. Physical examination shows prominent epicentral folds, microcephaly, a harsh systolic murmur heard best in the left fifth intercos

A micro deletion in the short arm of chromosome 5.
> This newborn has Cri-du-chat syndrome, which is caused by a
microdeletion in the short arm of chromosome 5.
> Individuals with this condition usually have moderate to severe intellectual disability and

Meiotic nondijucntion of chromosome 18 would result in ________________________, which is characterized by micrognathia, rocker-bottom feet, and clenched hands.

Edwards syndrome

Deletion of long arm of chromosome 22 will cause _______________________, which commonly results in recurrent viral and fungal infection (secondary to thrice alpaca), cardiac defects, abnormal facies with cleft palate, and hypocalcemia (secondary to parat

DiGeroge syndrome

A microdeletion in the long arm of chromosome 7 results in ___________________, which is characterized by an "elfin" facies, excessive sociability with well-dveloepd verbal skills, cardiovascular deficits, and hypercalcemia.

Williams syndrome

Imprinting of the long arm of chromosome 15 will result in either _________________ or __________________, depending on if the maternal or paternal chromosome was imprinted.

Angelman syndrome; Prader-Willi syndrome

Case: A 45 yo man with diabetes mellitus and uncontrolled hypertension comes to the ED because of tearing chest pain radiating to the mid scapular region. He smokes 1 pack of cigarettes daily. His blood pressure is 210/120 mmHg, pulse is 60/min, respirati

Hypertension
> This patient has an acute thoracic dissection (Sudden-onset, chest pain radiating to the back with mediastinal widening on chest radiography). Extension of aortic dissection to the root of the aorta may involve the ostia of the coronary art

__________________ is a catastrophic separation between the layers of the aorta. It is a intimal tear. Once breached, the blood enters the media layer & creates a new lumen.

> The media will continue to peel apart longitudinally Thus the aortic dissection can prorate into all the major vessels (carotids, renal arteries, iliac arteries, coronary arteries, pericardium)
Location
> The most common location is the first few centim

Case: A 31 yo man comes to the physician because of shortness of breath. he has a history of intravenous drug use in the past. Chest radiograph shows enlarged heart with a dilated right atrium. Examination shows jugular venous distention and pulsatile hep

Tricuspid regurgitation
> Intravenous drug users are at risk of infective endocarditis, most commonly caused by Staphylococcus aureus in this population. These microbes enter the right side of the heart via return from the right side of the heart via retu

Patients with _____________________ will have a wide pulse pressure and possibly have water hammer pulses and head bobbing on exam along with a descending decrescendo mumur

Aortic regurgitation

_______________________ most commonly occurs to rheumatic heart disease rather than IV drug use. Additionally, patients with mitral stenosis are unlikely to have JVD and a pulsatile liver (unless there is also right sided heart failure)

Mitral stenosis

Case: A 16 yo boy, recently migrated from India, comes to the physician because of increased fatigue, dyspnea and a blue tinge to the skin after exertion for the past 5 months. He reports having difficulty when playing sports with his friends. he often ge

Pulmonary hypertension
> The patient's cyanosis along with other symptoms of hypoxia, present later in life rather than early on. The congenital malformation that was present at birth is a VSD since echocardiography reveals a hole between ventricles. In a

__________________ is characterized by a downward displacement of the tricuspid valve leaflets into the right ventricle. It can cause
tricuspid regurgitation
and is commonly associated with lithium exposed to the fetus in utero.

Einstein's anomaly

_______________ is a condition of the pulmonary artery and aorta that can persist after birth. It is associated with a continuous machine-lied murmur heard at the left 2nd intercostal space during inspiration and expiration.

Patent ductus arteriousus (PDA)
> Uncorrected PDA can lead to Eisenmeger syndrome

Case: A 22 yo woman comes to the physician because of an upward ocular lens displacement. She has a history of 2 episodes of pneumothorax over the past 12 months. Physical examination shows inwardly grown ribs and sternum, producing a sunken appearance to

Dilated aortic root
>The patient presents with tall and thin stature, lens dislocation, arachnodactyly (positive thumb and wrist signs), high arched palate, precuts excavated, and history of pneumothorax, all suggesting
Marfan syndrome
as the underlying c

Coarctation of the aorta is associated with _______________________ and present with hypertension in the upper extremities and hypotension in the lower extremities. "Rib nothing" may be apparent on the chest x-ray.

Turner syndrome

____________________ has a early manifestation that usually occurs about 2-5 days after an MI

Acute fibrinous pericarditis

____________________ often presents with fever, malaise, and a new murmur. Blood cultures may be positive. There are no significant ECG changes with it unless the conduction system is also involved, perhaps from an abscess.

Infective endocarditis (IE)

___________________ is an important cause of persistent ST segment elevation after an MI. It is characterized by a paradoxical systolic budge palpable over the pericardial area. Ventricular aneursym can predispose to statin and thrombus formation.

Ventricular aneursym

Case: A previously healthy 39 yo woman comes to the physician because of a 2-month history of headaches, fevers, weight loss, and abdominal cramping worsened after meals. Her blood pressure is 188/110 mmHg. Physical examination shows diffuse purpuric less

Hepatitis B seropositivity
> Polyarteritis nodosa is an acute systemic vasculitis characterized by immune-complex-mediated necrotizing inflammation of the small and medium-sized vessels.
> Patients are usually young adults and symptoms develop over weeks

Case: A 55 yo man with history of hypertension, hypercholesterolemia, and diabetes comes to the ED because of shortness of breath, diaphoresis, and severe, left-sided, retrosternal, chest pain over the past hour. The pain radiates to the jaw and left arm.

It is located between the bundle of His and Purkinje fibers
> The patient is presenting with Right Bundle Branch Block (RBBB) post myocardial infarction.
RBBB is manifested by a wide splitting sound on auscultation
due to delayed right ventricular contrac

Damage to the ______________ results in delayed action potential conduction such as 1st, 2nd, or 3rd degree ____ block.

AV node; AV
It would NOT cause rabbit ears in V1 and V2 and wide splitting. The is characteristic of Right Bundle Branch Block (RBBB).

Damage to the ____________ may result in conduction abnormalities such as anterior or posterior fascicular block.

Purkinje fiber
> The Purkinje fibers conduct the cardiac action potentials through he inter ventricular septum to the ventricles.

Damage to the _________ would result in a slow heart rate.

Atria
> The atria conduct the cardiac action between between the SA anode and the AV node.

Case: A 25 yo woman at 14 weeks' gestation comes to the physician because of progressive shortness of breath over the past 2 weeks. A 3/6 ejection murmur is auscultated at the second left intercostal space near the sternum. The jugular vein is distended.

Fixed S2 splitting
> The pregnant woman has symptoms and sings of
heart failure
, including progressive shortness of breath, jugular vein distention, and hepatojugular reflex.
>During pregnancy, blood volume increases any pre-existing, but previously asym

An _________________ is heard in patients with prolonged hypertension (it is not associated with ASD)

Accentuated A2 sound

A
continous machinery murmur
at the left upper sternal border or second intercostal space beneath the clavicle with radiation to the back is characteristic of ___________________

Patent ductus arteriosus (PDA)

Pulsus paradoxus is seen in _______________

cardiac tamponade

Case: A 48 yo man comes to the physician for antihyperlipidemic therapy. He has an allergy to penicillin and sulfonamides. A fasting lipid panel shows: total cholesterol, 262 mg/dL; low-density lipoprotein (LDL), 188 mg/dL; high-density lipoprotein (HDL),

Triglyceride level
> Bil acid sequestrates have a minimal or even negative effect on triglyceride levels; in other words, in some cases they can raise
triglyceride levels
, modestly. This property makes them unique among antihyperlipidemic therapies. This

A ________________ can effetely lower a patient's high LDL levels by binding to negatively charged bile acids and salts in the small intestine, thus facilitating their excretion in the feces. This prompts the liver to divert cholesterol to form more bile

Bile acid sequestrate

Case: A 65 yo man comes to the physician because of a 6 month history of facial discoloration and photosensitivity. He has a history of atrial fibrillation, hypertension, and diabetes. (Bluish-grey discoloration)
What medication is most likely responsible

Amiodarone
> Amiodarone is used for atrial fibrillation. Long-term use and at doses greater than 400 mg/d increases risk. The pigmentation may be preceded by a photo allergic eruption. Pigmentation may slowly resolve months to years after withdrawal of th

What 2 side effects of Rosuvastatin?

Hepatotoxicity and myopathy
> Rosuvastatin is an HMG CoA reductase inhibitor used for the treatment of hypercholesterolemia.

What is 2 side effects of Ramipril?

Dry cough; angioedema
> Ramipril is an ACE inhibitor used for the treatment of hypertension.

What are side effect amlodipine?

Extremity edema, constipation, and flossing
> Amlodipine is a calcium channel blocker used in the treatment of hypertension

What is a side effect of Procainamide?

Drug induced lupus
> Procainamide is a class 1A anti arrhythmic medication

Case: A 50 yo woman comes to the physician because of lower extremity cellulitis. Physical examination reveals a first heart sound, and a second heart sound which changes in quality on deep inspiration at the left upper sternal border.
What is the most li

Increased return to the right hear during inspiration, which prolongs closure of the pulmonic valve.
> Closure of the mitral and tricuspid value mark the beginning of ventricular systole and produces the first heart sound or S1.
> The second heart sound,

A _______________ defect classically produced *fixed splitting of S2, which does not disappear with expiration.

Atrial septal (defect)

Patients with poor contractility of the left ventricle may present with a 3rd heart sound, which is highly specific for __________________. A 3rd heart sound occurs due to the filling of a ventricle that has not completely emptied due to poor contractilit

Heart Failure
> A 3rd heart sound would NOT be expected to change with inspiration and expiration.

Case: A 4-month-old infant is brought to the ED because of lethargy, poor feeding and clammy skin. She had a natural birth at home without any complications. However, she has not yet been evaluated by a pediatrician. Physical examination reveals a cyanosi

Total anomalous pulmonary venous return
> Total anomalous pulmonary venous return (TAPVR) is a congenital heart defect leading to cyanosis (blue babies) early in life. In TAPVR, the pulmonary vein drains in SVC instead of the left atrium. A pathognomic ra

Case: A 43 yo woman comes to the physician because of acute-onset of high fevers, lethargy and dyspnoea. Cardiac biomarkers are mildly elevated. An ECG shows normal sinus rhythm. A clinical diagnosis of viral myocarditis is made.
What is the most likely c

Coxsackie B virus
> Coxsackie B virus is the most common cause of viral myocarditis

_______________ is the most common cause of the common cold

Coronavirus

____________________ virus causes croup in young children

Parainfluenza

Echovirus causes ___________________

aseptic meningitis

Case: A 63 yo woman comes to the physician because of shortness of breath and an episode of near syncope. She is a chronic smoker and was recently diagnosed with an inoperable breast cancer. She has had an 18 kg (40 lb) weight loss over the past year. Her

Cardiac tamponade
> This patient has tachycardia, hypotension,
pulsus paradoxes (significant drop in blood pressure during inspiration), distended neck veins, muffled heart sounds, low voltage QRS complexes and globular cardiac show; these features sugges

________________________ is characterized by thickens, fibrotic and calcification pericardium as a result of chronic inflammation or scarring. As a result, cardiac expansion is impaired and a diastolic dysfunction ensues, more significantly affecting the

constrictive pericarditis
> Tuberculosis is the most common cause of constrictive pericarditis worldwide, while most cases in the US are idiopathic, or secondary to scarring from previous open-heart surgery or period chest irradiation, or pericardial meta

Case: A 72 yo man with history of hyperlipidemia and HTN presents to the ED with cough and shortness of breath the that bene worsening over the past 24 hours. On examination, he is afebrile, has blood pressure of 10/60 mmHg, heart rate of 110 BPM and O2 S

Pulmonary edema
> This patient is most likely suffering from flash pulmonary edema. he has multiple cardiac risk factors including advanced age, hypertension, and hyperlipidemia. He has a new systolic murmur that is loudest at the apex, most likely repres

Pericardial effusion is milky to manifest with Beck's raid of ________, ________, and ________

Distant heart sounds, hypotension, and distended neck veins.

Dual antiplatelet therapy is used for ________________________ in patients who had a stroke while on a single antiplatelet agent. However dual anti platelet therapy is not indicated for primary stroke prevention in atrial fibrillation or atrial flutter.

Secondary stroke prevention
Ex. Clopidogrel and aspirin
Ex. Dipyridamole and aspirin

A EKG shows F-waves (saw tooth appearance) and is diagnostic for __________________

Atrial flutter
> Future stroke risk in patients with atrial flutter is high. The CHA2D2-VASc score is used to predict future stroke risk and whether long-term management should be with antiplatelet therapy or anticoagulation.
> Patients with a CHA2DS2-VAS

________________________ is highly associated with an increased risk of developing Alzheimer's dementia.

Down Syndrome
> The extra copy of chromosome 21 confers an increased risk of accumulating the amyloid precursor protein responsible for cereal plaque development in Alzheimer's dementia.

The most common congenital cardiac defect is Down sydrnome is ___________

Endocardial cushion (atrioventricular septal defect).
> Gestational testing for this syndrome includes testing for decreased serum PAPP-A increased free beta-hCG.
> Ultrasound testing in the first trimester will show increased nuchal translucency, and sec

Case: A 32 yo man comes to the ED because of extreme pain in his fingers that started two weeks ago. The pain is constant, even at rest, and nothing alleviates it. His past medical history is unremarkable his vital sings are within normal limits. Addition

Thromboangiitis obliterans
> This is a typical presentation of aprons with a history of heavy smoking that has Buerger diseases, also known as thromboangiitis obliterans.. This disease is characterized by acute and chronic inflammation of small and medium

Microscopic polyanittis is a cnecrotizing vasculitis commonly involved in the lung, kidneys, and skin. It can present with palpable pupura and is a_________________ positive.

Myeloperoxidase.

What disorder?
Patient can present with unilateral headache, visual abnormalities, facial pain especially at the temporal area where the temporal artery courses?

Gain cell arteritis (aka temporal arteritis)
> Giant cell arteries is often associated with polymyalgia rheumatic.

_________________ disease is aurally seen in young children of Asian descent. These patients can present with high fever, conjunctival injection, strawberry tongue, desquamating rash, cervical lymphadenopathy (unilateral) and red/swollen hands and feet. U

Kawasaki

_______________________ also known as "pulseless disease" is seen in Asian females <40 years old presenting with fever, night seater, arthritis, myalgia, skin nodules, and ocular disturbances. They will have decreased upper extremity pulses compared with

Takayasu arteritis
> The pathological mechanism behind this disease is granulomatous vasculitis of medium and large arteries (especially aortic arch) that leads to narrowing.

Case: An infant is born at 38 weeks' gestation and has low Apgar scores following birth. ECG demonstrates very poor right ventricular function with downward displacement of the tricuspid valve.
Which one of the maternal medical complications most likely t

Long-standing bipolar disorder
> The lesion described on the ECG is suggestive Ebstein's anomaly. This is classically associated with lithium use, therefore, a maternal
long-standing bipolar disorder
is the most likely contribution in this case.
> Ebstein

_________________________ are contraindicated in prelacy because of the effects upon fetal teeth development and inhibition of bone growth

Doxycycline and the tetracyclines

Case: a 56 y/o African-American man with COPD and CAD comes to the physician for a routine examination. His average diurnal ambulatory blood pressure is 148/92 mmHg. Lab studies show creatinine level of 3.2 mg/dL and a potassium level of 5.6 mg/dL.
What o

Amlodipine
>
Amlodipine
is used alone or in combiatnion with the medications to treat high blood pressure. Also, in patients with CAD (coronary artery disease), Amlodipine is used to reduce the risk of hosptilzaiton for anger and to reduce the risk of a c

What type of blood pressure emaciation is contraindicated during pregnancy because it can cause defects in kidney function?

Angiotensin-converted enzyme (ACE) inhibitors

___________________ is an antithyroid medication that can be used during pregnancy

Propylthiourcail
> Propylthiourcial is recommended during the first trimester, as there is risk of liver damage to the fetus with use of methimazole during this period. Patients should be maintained on methimazole for the remainder of the pregnancy.

What anticoagulant has a contraindications during pregnancy as it causes mental retardation, microcephaly, and the complications of the fetus?

Warfarin

Q fever is caused by _____________________. This disease is character by pneumonia.

Coxiella burnetii

Typhoid fever is caused by ____________________. Fever, diarrhea, headache, and red abdominal spots characterized this disease.

Salmonella typhi

_______________ is caused by paramyxovirus. Throat cultures would not produce bacterial outgrowth.

Croup
> Croup is common in children and is characterized by a deep, barking cough.

______________________ infection results in
rheumatic fever
. Symptoms typically develop 2-4 weeks after a throat infection and include: fever, multiple painful joints with the joints affected changing wth time, involuntary muscle movements (Chorea), and

Streptococcus pyogenes

Case: A 66 yo woman comes to the physician because of increased fatigue and dyspnea upon exertion for the past 2 weeks. She also reports two episodes of fainting while working in her garden. her ital signs are all within normal limits. An ECG performed la

Paradoxical splitting
>The patients ECG finding incite aortic stenosis. In aortic stenosis, P2 is heard before A2 because the pulmonic valve closes before the aortic. This happens because the time its takes to eject boo across a stenotic valve is greater

A split during inspiration and expiration occurs in _____________, where there is left-to-right shunt. The right veneticualr ejection time will always be delayed and the pulmonic valve will always close after the aortic valve due to increased blood volume

ASD (Atrial septum defect)

A _______________________ is heard in aortic regurgitation. It has a high-pitched/blowing nature caused by the turbulence of blood returning to the left ventricle through the insufficient aortic valve. It is best heard at the apex.

A diastolic decrescendo murmur

A harsh ____________________ murmur is heard in VSD, and is louder at the tricuspid area.

Holosystolic

A split during inspiration is a ______________ finding. During inspiration there is increased venous return in the heart, therefore, increased right ventricular ejection time and delayed closure of the pulmonary valve.

Normal

Case: An 8 month-old infant is admitted to the hospital for treatment of a viral upper respiratory infection and dehydration. The infant has a history of multiple admissions for severe gastroenteritis, failure to thrive secondary to poor/difficult feeding

A single common vessel draining both heart ventricles
> The vignette describes an infant with DiGeorge syndrome caused by a microdeltion of the long arm of chromosome 22 (22q11). This condition is characterized by abnormal facies, thymic and parathyroid a

A single palmar crease is a finding associated with _______________________

Trisomy 21 (Down sydrnome)

Case: a 67 yo diabetic woman comes to the physician because of a squeezing retrosternal chest discomfort over the past 3 weeks. The episodes of chest pain last about three minutes, are precipitated by walking, and are received by rest. She has a history o

Stable aninga pectoris
> The chest pain in this patient is retrosternal in location, precipitated by physical exertion and received by rest, suggesting concern for obstructive coronary artery diseases, or 'stable angina'. It is important to remember that

_________________ is describes chest discomfort with one or more of the following characteristics:
1) Occurrence at rest or with minimal exertion
2) Increasing in severity or frequency; accelerating pattern
3) Episodes of chest discomfort > 20 minutes in

Unstable angina

What does a aortic stenosis murmur sound like?

There is a fourth heart sound heard in late diastole (just before the first heart sound). This is caused by the increased left ventricular wall thickness and stiffness.
> It is caused by calcification of the aortic valve leaflets.

__________________ manifests with sudden-onset, severe, tearing chest pain that typically radiate to the back. The pain is usually unremitting and must be treated with stringent blood pressure control and emergency surgery (in the case of a proximal type

Aortic dissection

Case: A 45 yo healthy potential kidney donor is admitted to the hospital for preoperative cardiac testing. Inhaled/exhaled breath analysis shows oxygen consumption of 200 mL/min. Central venous oxygen content is 8 mL oxygen/100 mL, pulmonary artery oxygen

4 litres/min
> Cardiac output can be calculated by assessing the ratio of the total oxygen consumption to the arteriovenous oxygen difference across the lungs. In this case, the arteriovenous difference would be measured across either side of the lungs th

Case: A full-term male newborn is examined shortly after birth. He has cyanotic discoloration of the lips and mouth. The pregnancy and labor were uneventful. Arterial oxygen starvation is 70%. Arterial partial pressure of oxygen is 35 mmHg after administr

Transposition of great arteries
> The most common congenital heart disease presenting with cyanosis within the first day of brith is transposition of great arteries (TGA). In this malformation, the aorta originates from the morphologic right ventricle and

_____________________ is comprised of the following anomalies; pulmonary stenosis; overriding aorta, ventricular septal defect, and right ventricular hypertrophy.

Tetralogy of Fallot
> Tetralogy of Fallot with severe pulmonary stenos results in cyanosis early after birth because blood flows preferentially through the path of least resistance: a right-to-left shunt through the ventricular septal defect.
> Right-to-l

Case: A newborn boy is seen by the pediatric cardiologist 2 days after birth because of persistent cyanosis, dyspnea and two seizure episodes. He has low set ears, bounding peripheral pulses, and a single S2 is heard on heart auscultation. Chest X-ray rev

Truncus arteriosus
> The newborn baby has DiGeorge syndrome, a chromosomal abnormality caused by 22q11 deletion. She patients present with: thyme aplasia/hyposplasia, hypocalcemia, abnormal facies, cardiac abnormalities, and cleft palate (CATCH-22).
> Rad

Tetraoloyg of Fallot has the same clinical presentation, as it is a cyanotic condition and it is also very common in patients with DiGeorge Syndrome. However, X-ray classically shows a ________________ heart

Boot-shaped

The common radiology finding with Transposition of the Great Vessels is? (Common is patients with DiGeorge Syndrome).

egg on a string"-shaped heart
> Chest X-ray may show absence of the main pulmonary artery, increased vascular marking, and single round S2 as the aorta will be superior to the pulmonary artery.

________________________ has the same clinical presentation as it is a cyanotic condition. There is no tricuspid valve, the pulmonary artery and valve are underdeveloped, and the right ventricle is hypo plastic. Since there is little blood flow across the

Tricuspid atresia

Case: A 5 yo girl is brought to the physician because of an unusually wide neck and swelling on her hands and feet. Karyotyping shows monosomy X and 45X. Physical examination shows a mild aortic stenosis.
What is the the congenital condition that is most

Preductal coarctation
The stem of the question describes Turner syndrome, which is associated with
preductal coarctation
of the aorta (infantile type).
> There is aortic stenosis proximal to insertion of the ductus arteriosus. Physical characteristics inc

Patent ductus arteriosus is associated with ______________________

congenital rubella

_________________ (vessel disorder) is NOT associated with any congenital diseases

Transposition of the Great Vessels
> However, it is associated with maternal diabetes

Case: A 20 yo man is brought to the ED because of SOB. He is tachypneic. Chest auscultation revels clear lungs bilaterally. An arterial blood gas shows PO2 of 50 mmHg. The patient has an alveolar PO2 of 100 mmHg, and supplemental O2 does not improve the p

Complete right-to-left shunt
> A
complete right-to-left shunt
implies that blood is bypassing the lungs and traveling form the right heart to the left heart without being exposed to oxygen. Therefore, the alveolar PO2 will be normal, but the arterial bloo

A ___________________ means that gas exchange between the alveolus and capillary is reduced, but not completely absent. This will create an A-a gradient as alveolar O2 levels will be normal, but arterial O2 levels will be reduced. This hypoxemia will corr

Partial diffusion abnormality

______________________ will cause hypoxemia with an elevated A-a gradient, but it will correct a inspired O2 levels increase

Ventilation-perfusion mismatch

Case: A 29 yo man comes to the physician because of recurrent palpitations, lightheadedness, blurred vision, and feeling faint. During episodes, he appears pale and diaphoretic, and experiences loss of postural control without losing consciousness. Each e

Torsades de pointes
> The patient's recurrent episodes of light-headedness with loss of postural control without losing consciousness are consistent with pre syncope (as opposed to syncope, which is associated with transient loss of consciousness and post

Case: A 26 yo woman at 38 weeks gestation comes to the hospital in labor. She has been having an uncomplicated pregnancy so far. All her prenatal testing showed no abnormalities. In utero, the fetus has a speckled type of circulation of blood from the mot

Ductus Arteriosus
> The fetal circulation is unique as it uses certain structures to shunt blood bypassing other structures. Blood will flow form the RA to the RV and then to the pulmonary artery, where through the
ductus arteriosus
it will enter the desc

_____________ is a structure that shunts blood from the umbilical vein to the IVC bypassing the liver.

Ductus venosus

Case: A 50 yo woman is brought to the ED because of confusion over the past few days. She has 1 week history of diarrhea, anorexia, and vomiting. She had CHF and takes digoxin. She was prescribed an antiarrhytmic drug for palpation 10 days ago.
Which of t

Quinidine
> This patient was prescribed quinidine.
Quinidine
, amiodarone, verapamil, and spironlocatone all displace digoxin from its benign site resulting in digoxin toxicity.
> Digoxin toxicity can cause CNS symptoms such as confusion, halluncintaiton,

______________ is most commonly used in patients with supraventicular tachycardia. It works by decreasing sinoatrial and atrioventricular node activity.
It should be avoided in patients with asthma

Adenosine

____________________ is a class IB anti-arrhrthemic drug. It is the drug of choice in arrhythmia post myocardial infarction, in arrhythmia due to digoxin toxicity, and following cardioversion.

Lidocaine

______________ is a class III anti-arrhymic drug. It is a potassium channel blocker that increase action potential duration and the end resting potential. It also blocks beta-1 receptors resulting in bradycardia and decreased atrioventricular conduction.

Sotalol

Case: A 2-day-old newborn girl is evaluated from failure to pass meconium. She was delivered vaginally at 39 weeks. APGAR score at delivery was 9 and 9 at 1 and 5 mins. She had no prenatal care. Her temperature is 41.5 C (107 F), pulse is 140/min, respira

Ventricular septal defect
>
VACTERL
is a condition that is associated with a group of brith defects that include
-
V
ertebral anomalies
-
A
nal atresia
-
C
ardiac defects such as ASD and VSD
-
T
rachesophageal fistulas
-
E
sogphageal atresia
- *R*enal and

Coarctation of aorta is seen in _______________

Turner syndrome

____________ is seen in Potter sequence

Pulmonary hypoplasia
P
ulmonary hypolasia
O
ligohydraminos
T
wisted face/skin
E
xtremity defects
R
enal failure (in utero)
Potter sequence is the atypical physical appearance of a baby due to oligohydramnios experienced when in the uterus. Oligohydramnios

Case A 40 yo man comes to the physician for a routine examination. There is an early systolic ejection clock on cardiac ausculatiaton and blowing diastolic murmur over the third left intercostal space close to the sternum. S1 and S2 heart sounds are norma

bicuspid aortic valve without calcification
> A bicuspid aortic valve is one of the most common congenital heart malformations in adults. It is typically associated with an early systolic ejection click.
> There is often some degree of aortic regurgitatio

Case: A 69 yo man with severe left side heart failure on combination drug therapy comes to the ED because of SOB, palpitations, nausea, and visual hallucinations. He sees halos around objects and a yellow-green color distortion in his vision. Examination

Displacement form tissue binding sites
> The mechanisms of toxicity from verapamil interaction is
displacement
. The two drugs share tissue protein-binding sites and, since the therapeutic window of digitalis is so small, this displacement can raise the s

Case: a 66 yol woman from Nicaragua comes to the physician because of SOB. She has atrial fibrillation and a heart murmur. The appearance of red flush appearance on both sides of the face.
What is most likely cause of her symptoms?

Mitral stenosis
> The patient has a
malar flush
. This is a pinkish-purple discoloration of the cheeks classically associated with mitral stenosis due to the vasodilatory effects of CO2 retention. The malar flushing tends to correlate with the severity of

A 60 y/o man comes to the ED because of severe abdominal pain, nausea, and vomiting. Lab studies show:
PaO2 90 mmHg; pH 7.35; PaCO2 27 mmHg; HCO3 8 mEq/L; glucose 80 mg/dL; urine dipstick did not detect ketones; anion gap 25 mEq/L
What is the most likely

Ruptured abdominal aortic aneurysm (AAA)
> The patient's lab values real an academia that is primarily of metabolic organs given the patient's bicarbonate level. Additionally, low PaCO2 indicates the patient's respiratory system is trying to compensate
>

Vomiting results in a __________________ with raised pH and bicarbonate due to the loss of HCL from emesis

Metabolic alkalosis

COPD results in a compensated __________________ due to chronic CO2 retention

Respiratory acidosis

Case: A 14 yo boy, who recently migrated from Thailand, comes to the physician for physical evolution. His family history is notable for sudden cardiac death of his paternal uncle and first cousin, at the ages of 23 and 16, respectively. An electrocardiog

...

Case: A 60 yo man is admitted to the hostpialf owl lowing a hypertensive crisis. His blood pressure is 180/120 mmHg. he has no chest pain, headache, or blurry vision.
What substance is vasodilator the may help in this scenario?

Sodium nitroprusside
> The endothelium-derived relaxing factor has since been recognized as nitric oxide (a vasodilator).
Sodium nitroprusside
is metabolized in the body-forming nitric oxide.
FYI: Thromboxane A2 does directly affect the vascular diameter,

Case: A 48 yo man with mitral regurgitation (MR) comes to the physician for a routine examination. The best position to auscultate for MR is usually at the apex.
What option most accurately corresponds to the location to auscultate?

Left 5th intercostal space
The apex of the heart is usually found at the left 5th intercostal space along the midclavilualr line. This is usually the best location for asculatating the mitral valve.
> The left 2nd inter cost space, just lateral to the lef

Case: A 69 yo man, who has a history of diabetes mellitus type 2, hypercholesterolemia and hypertension, comes to the ED 8 hours after the onset of acute chest pain, shortness of breath, and diaphoresis, which lasted for approximately 45 minutes. Since th

Right coronary artery
> This patient is presenting with sympotmic sinus bradycardia following acute myocardial infarction as evidenced by the ST elevations in leads II, III, and aVF.
> Sinus bradycardia following MI suggests dysfunction of the sinoatrial

_____________________ is associated with a progressively increasing PR interval followed by as dropped QRS complex

Second-degree Moritz I block (or Wenchkebach AV Block)

_______________________________ is characterized by constant unchanging PR intervals (usually normal in duration but can be prolonged) followed by the block of one or more P waves that fail to conduct to the ventricle.

Second-degree heart block, Mobitz type 2
> Moritz type 2 is a high degree AV block with the potential for progressing to complete heart block

Case: A 60 yo woman comes to the physician because of a headache over the past 24 hours. She has no altered conciseness, change in version, nausea, or pulsating. The headache is localized to the right side of the head just postern-lateral to the orbit. Ex

Granulomatous change
> This patient is likely to have temporal arteritis, a granulomatous inflammation of the temporal artery.
> Patients present with pain over the temple and can have a bulging temporal artery. Pain is exacerbated on palpation of the art

Case: A 62 yo man comes to the physician because of difficulty breathing for the past 3 months. He was diagnosed with hypertension 10 years ago. he also had a right sided, myocardial infarction 1 year ago. The patient also notes that he has not been havin

Decreased Heart contractibility
> The patient's presentation is consistent with congestive heart failure (CHF). In early stages of CHF the decreased cardiac output (CO) will activate the RAAS and the sympathetic nervous system which will cause systemic va

________________ is caused by left heart failure which is due to decreased myocardial contractility. it is not the main cause of edema, rather it is a sequela of events due to decreased contractility in the left ventricle.

Pulmonary congestion

________________ are associated with acute inflammatory states and bacterial infections.

Neutrophils
> Neutrophils are typically seen in conditions of exudative purulent fluid drainage.

Sheets of small cells are non-specific, but they suggest malignancy as in the case of _________________

small cell lung cancer

____________________ are associated with chronic inflammation or viral infection. They are generally clear and non-purulent in appearance

Lymphocytic infiltrates

Case: A 26 yo woman at 38 weeks of gestation comes to the hostpial for labor. She has been having an uncomplicated pregnancy so far. None of her prenatal testing shows any abnormalities. While in utero, the fetus has a specialized type of blood circulatio

Ligamentum venosum
> The fetal circulation is unique as it makes use of certain structures to shunt blood, bypassing other structures. There are three important shunts in fetal circulation, the ductus arteriosus, foramen ovale and ductus venous.
> The
lig

Case: A 63-year-old man with a history of chronic poorly controlled hypertension is brought to the ED because of sudden-onset, tearing, chest pain with radiation to the back and abdomen. He has prominent shortness of breath. His blood pressure is 170/70 m

Left renal artery
> This patient is presenting with aortic dissection of the proximal aorta followed by extension of the dissection long the aorta into the abdomen, causing this patient's acute onset abdominal pain and unconsciousness. Longstanding hypert

The extension of aortic dissection is most likely to propagate long the _________________ side of the aorta.

Posterolateral side
(Inferior mesenteric artery, superior mesenteric artery, anterior branch of aorta are all less likely to be involved in aortic dissection)

Case: An otherwise healthy 35-year-old man is admitted to the hospital for testing as part of a clinical research study. Inhaled/exhaled breath analysis shows an oxygen consumption of 350 mL/min. Mixed venous oxygen content is 12 mL oxygen/100 mL, the pul

63 mL
> Cardiac output can be calculated by assessing the ratio of the total oxygen consumption to the arteriovenous oxygen difference across the lungs. In this case, the arteriovenous difference would be measured across either side of the lungs through w

Case: A 5 year old girl was borough to the ED because of bluish discoloration of the lips and mouth after playing with her friends. An echocardiogram shows a defect in the upper portion of the ventricular septum and a slightly enlarged right ventricle
If

Pulmonary vascular hypertrophy with suit reversal leading to congestive heart failure
> The girl was born with ventricular septal defect (VSD) that was not corrected at birth. In VSD there is a shunt of blood from the left ventricle to the right ventricle

A 5-yo boy is brought to the physican by his mother because of abdominal pain, fever and rash over the past 5 days. The boy appears irritable. His temperature is 103.1F. Physical examination shows bilateral conjunctival infection, redness of the oral muco

Aspirin and intravenous immunoglobulin
> Mucocutaneous lymph node syndrome (Kawasaki's ideas) is a self-limiting medium-vessel vasculitis that commonly occurs in children under 10 years-old. Although it is self-resolving, severe cardiac complications may

_____________________ is first-line treatment for infection with Borrelia bugodorferi *Lyme disease), which typically presents with a target shaped rash and frequently associated with joint pain and malaise. However, doxycycline is not routinely used in y

Doxycycline
> However, doxycycline is not routinely used in young children under 8 years of age unless absolutely necessary due to tooth discoloration.

_____________ is used to treat strep throat, which may resent with swollen red tongue and oropharynx.

Amoxicillin

Acetaminophen is an antipyretic used to symptomatically control fevers in young children. Patients with Kawasaki's disease may be given acetaminophen to reduce fever and improve patient comfort, however, _____________ is still required to prevent complica

Aspirin

Case: A 3 month old infant is brought to the physician because of difficulty feeding, failure to thrive, and episodes of blush pale skin during crying or feeding. Physical examination reveals a harsh systolic ejection murmur that is heard over the pulmoni

Anterosuperior displacement of the infundibular septum
> A boot-shaped heart is a characteristic of Tetralogy of Fallot, which is caused by
anterosuperior displacement of the infundibular septum
. The four cardiac defect associated with this condition are

Failure of the aorticopulmonary septum to spiral refers tot he mechanism that leads to _____________________ transposition of the great vessels. This is cyanotic heart disease the tis not compatible with life unless a shunt is present to allow the mixing

Transposition of the Great Vessels.

Failure of the aorticopulmonary septum to divide describes persistent ____________________, which results in right ventricular hypertrophy due to the persistence of the right to the left shunt. A continuous machine-like murmur is heard on cardiac examinat

ductus arteriosus

_________________________ refers to obstruction of outlaw form the right ventricle (RV) within the body of the RV.

Infundibular pulmonary stenosis (IPS)
> Underdevelopment of conus arteriosus of the right ventricle gives rise to
infundibular stenosis
.

Case: A full-term male newborn is examined shortly after brith. He has up-slanting palpebral issues, epicanthic folds, flat nose bridge, and low-set ears. Precnacy and delivery were uncomplicated. The mother is 38 years old and test negative for group B s

Complete atrioventricular septal defect
> The physical feature of the infant most closely match those of Down syndrome (trisomy 21). On prenatal screening, Down sydrnome fetuses tend to be associated with an elevated total beta-human chorionic gonadotropi

______________________ is a complete absence of the tricuspid valve with no communication between the right atrium and the right ventricle.

Tricuspid atresia
> Tricuspid atria's is most often associated with
atrial septal defect (patent foramen ovale)
, which allows the blood to exit the right atrium so that deoxygenated blood will enter the left atrium. The infant's presentation is not consi

________________ is characterized by the appeal displacement of the tricuspid valve, resulting in a decreased right ventricular volume and atrialization of the right ventricle.

Epstein anomaly
> Ebstein anomaly is usually associated with lithium usage during pregnancy.

Case: A 68 yo women with hypertension comes to the physician because of gingival hyperplasia. She takes a blood pressure medication though she is unable to recall its name presently.
What medication is most likely responsible?

Nifedipine
>
Nifedipine
is a dihydropyridine calcium-channel blocker used to treat hypertension. Side effects include reflex tachycardia, pedal edema, and gingival hyperplasia. Other medications that can cause gingival hyperplasia include
phenytoin
and
cy

What are two side effects of lisinopril?

Renal dysfunction; a bradykinin mediated dry cough
> Lisinopril is an angiotensin-coveting enzyme (ACE)-inhibitor

_________________ is a mineralocorticoid antagonist that can cause hyperkalemia and gynecomastia

Spironolactone

___________________ is a cardiac glycoside that blocks the sodium/potassium-ATPase channel. It has a narrow therapeutic index and is associated with adverse effects when plasma levels exceed this range. Adverse effects include nausea, cardiac arrhythmia,

Digoxin

__________________ is a beta-adrenoceptor antagonist. Its most common side-effects include lethargy, bronchospasm, bradycardia, and hypotension.

Carvediolol

A 67-year old male patient comes to the ED because of bilateral ankle edema. Examination shows elevated jugular venous pressure at 7 cm above the sternal angle and there are large V-waves. A soft pan systolic murmur is audible at the left sternal edge on

Tricuspid regurgitation
>
Tricuspid regurgitation
leads to an elevated JVP with large V-waves and a pan systolic murmur at the left sternal edge. Other features of tricuspid regurgitation are pulsatile hepatomegaly and left parasternal heave.

_________________ causes a mid-diastolic murmur at the apex and severe cases may have secondary pulmonary hypertension (a cause of tricuspid regurgitation)

Mitral stenosis

________________ causes a pan systolic murmur at the apex, which radiates to the axilla.

Mitral regurgitation

____________________ cases an ejection systolic murmur that radiates to the neck

Aortic stenosis

The left circumflex artery supplies the _______ and/or _______________ in a minority of individuals. In the majority of individuals, the SA node is supplied by a direct branch of the _______________________

SA; atrioventricular node; proximal right coronary artery

Case: An 8 yo boy is brough tot eh ED by his father because of abomdinal pain. He had a recurrent upper infection a few weeks ago. He is afebrile and his vital signs are within normal limits. Physical examination shows mild diffuse abdominal tenderness an

Immune complexes of IgA deposited in vessels
> This is a classic presentation of allergic purpura (Henoch-Schonlein purr, HSP), a small-vessel vasculitis caused by the
depostiion of IgA complexes
. It commonly affects children and often follows an upper r

_________________________ is a granulomatous vasculitis characterized by eosinophilia. It commonly presents with asthma, sinusitis, skin lesions, and peripheral neuropathy.

Polyarteritis with lung involvement (Churg-Strauss syndrome)
> Churg-Strauss is associated with elevations of p-ANCA.

____________________ can develop as a complication of streptococcal infection. Immune complexes formed during the infection can then become lodged in the glomerualuar basement membrane, causing destruction.
This damage to the glomerulus leads to hematuria

...

______________________ can also cause a purpuric rash especially on the extremities, but one would expect to see low platelet and would not see the signs such s abdominal pain.

Idiopathic thrombocytopenia purpura
ITP is caused b the formation of autoantibodies against platelet surface antigens, which ultimately leads to opsonization and destruction of platelets. This leads to an increased tendency to bleed, manifesting as bruise

What is the pentad of TTP? The cause?

TTP pentad
1) Thrombocytopenia
2) Microangiopathic hemolytic anemia
3) Neurological symptoms
4) Kidney failure
5) fever
TTP is caused by inhibition of
ADAMTS13
, a metalloproteinase responsible for the breakdown of von Willebrand factor (vWF).
> Thwne ADA

Case: A 48 yo woman comes to the ED because of severe chest pain that radiates down her let arm. She has felt a similar pain before, but it received itself after time. The pain is described as crushing dip to the sternum. She has a history of hypertension

Left 5th and 6th intercostal spaces midclavicular
> This patient, who has presented a few days after MI, is likely o have a mitral regurgitation secondary to papillary muscle rupture that can occur 2-7 days post-MI. A mitral valve murmur is best appreciat

Case: A 62 yo man with idiopathic pericarditis comes to the physician because of confusion and complaining of chest pressure. Physical examination shows hypotension, jugular venous dissension and muffled heart sounds on auscultation. EKG shows alternating

Between the epicardium and the parietal pericardium
> The patient in this vignette has presented with cardiac tamponade (often presented as Beck's triad of hypotension, jugular venous distention, and muffled heart sounds and electrical alternates on EKG (

Case: A 68 yo man comes to the physician because of a sensation of food getting stuck in the his chest when swallowing. The sensation is at the mid-sternum, and it is most pronounced when eating solids, rather than liquids. he does not have difficulty ini

Compression by an extra-esophageal source
> This patient's symptoms are most consistent with dysphagia , which may arise secondary to a wide variety of causes including esophageal dymotiblity, infection, autoimmune esophagitis, and mechanical obstruction.

_____________________ is an inflammatory condition of the esophagus that typically presents during the second or third decade of life. It often occurs chronically and patients present with extensive histories of difficulty swallowing and feeling as though

Eosinophilic esophagitis

Case: A 57 yo old man, recently migrated from India, comes to the physician because of coughing, difficulty breathing, and trouble swallowing. His vital signs are within normal limits. Physical examination reveals three 2-cm gummas on the patient's face a

Obliteration of the vasa vasorum
> This patient suffers from tertiary syphilis, a sexually transmitted infection with Treponema palladium. Untreated primary infections an lead to secondary, and then territory sysphilis after many years.
> Tertiary syphili

Lymphocyte, macrophage, and ___________________ infiltration describes the pathological mechanism of inflammatory abdominal aortic aneurysm

Giant cell

Bicuspid aorta

Calcification of a _______________ can lead to aortic stenosis in patients of a younger age. It can cause thoracic aneurysm.

_____________________ is seen in cases where there is abrupt and severe elevation of blood pressure. Microscopic findings is the characteristic "onion skin" thickening of the arterioles

Hyperplastic arteriolosclerosis

Case: A 14 yo boy, who recently migrated from Thailand, comes to the physician for physical evaluation. His family history is notable for sudden cardiac death of his paternal uncle and first cousin, at the ads of 23 and 16, respectively. An ECG performed

An exaggerated split during inspiration
> This question requires knowledge of several pathognomonic findings. The patient's age, origin, family history, and mode of inheritance indicates a genetic disorder, in this case Brugada Syndrome.
> Brugada Syndrom

Case: A 52 yo man comes to the physician because of difficulty breathing for the past 3 months. He was diagnosed with hypertension 3 years ago but has not been compliant with his medication. The patient also notes the use of three pilots in order to sleep

Increased venous return
> The patient's presentation is consistent with heart failure due to left ventricular abnormality (S3 gallop on the heart apex and displaced PMI).
> Orthopnea is defined as the shortness of breath while in a supine position. Becaus

Increased heart contractility
is a compensatory mechanism in the
early stages
of ____________ that will maintain CO (Cardiac Output).

Heart failure

A 24 yo man is brought to the ED after a motor vehicle accident. He was not wearing a seatbelt and was ejected from the passenger seat. He was found 20 ft away from the accident. His blood pressure is 88/55 mmHg, pulse is 110/min, respiration's are 22/min

Proximal defending aorta rupture
> The patient was involved in motor vehicle accident, in which many structures can be damaged because of high impact. patients that go through rapid deceleration or direct blunt trauma to the chest are highly susceptible t

Myocardial rupture would most likely lead to immediate death. If the bleeding is contained by the pericardium, the patient will present with the classic ______________ (distant heart sounds, distended neck veins and hypotension).

Becks Triad

Esophageal rupture, also known as ___________________, is caused by blunt trauma during endoscopic procedures, or during severe retching. X-ray will reveal pneumomediastinum/pleural effusion.

Boerhaave Syndrome

A 54 yo man develops premature ventricular contractions that are unresponsive to electrical cardioversion. He is administered propafenone but dies soon after.
What best describes the effect propafenone has on the ventricular myocardium?

QT prolongation
> Propafenone may cause a condition that affects the heart rhythm (QT prolongation). QT prolongation can rarely cause serious (rarely fatal) fast/irregular heartbeat and other symptoms (such as severe dizziness, fainting) that need medical

Class IB agents, such as __________, block inactivated Na+ channels.

Lidocaine

Class III, and to lesser extent class IA anti arrhythmic agent, ____________________________

block K+ channels and prolongrepolarization.

Digoxin increases myocardial contractibility by ______________________________

blocking Na/K ATPase and increasing intracellular Ca2+

Slowing conduction through the sinoatrial SA) and atrioventricular (AV) node is a property of ____________ such as atenolol

beta-blockers

Case: A 26 yo woman who recently immigrated to the USA from Asia comes to the physician because of palpitations and exertion dyspnea. There is a mid-diastolic mummer with a
loud first heart sound
on cardiac auscultation.
What is the most likely diagnosis?

Mitral stenosis
> Mitral stenosis is often secondary to childhood rheumatic fever. While rare in developed countries, it remains commonplace in developing nations. The presence of a loud first heart sound, opening snap and a diastolic mummer are suggestiv

___________________ causes pansystolic murmur

Ventricular septal defect

Case: A newborn is examined shortly after birth. She appears cyanotic. A chest X-ray shows a "boot-shaped" heart, and a holosystolic murmur is audible at the left lower sternal border.
What finding is also likely in this patient?

Right ventrciular outflow tract obstruction
> A cyanotic neonate with a boot-shaped heart and a holosystolic murmur loudest at the left sternal border is likely to have tetralogy of Fallot (TOF). TOF consists of:
1) Right ventricular hypertrophy (accounts

Case: A previously healthy 65 yo woman is brought to the ED because of acute stroke. She is afebrile, BP is 180/100, HR 75, and RR 14. Physical examination shows no heart murmurs, unilateral lower extremity edema and redness. Cardiac monitoring shows norm

Patent foramen ovale
> Paradoxical embolic stroke occurs when venous thromboses avoid clot-capture by the lungs and enter the pyemic circulation by way of a Right to Left shunt through a
patent foramen ovale (PFO)
. PFO does not usually produce symptoms a

A 28 yo G3P1 woman, at 34 weeks' gestation, comes to the physician for her prenatal visit. She has been experiencing normal pregnancy so far, without ay complications and has not missed any appointments. She reports that she has been having severe headach

Hydralazine
> The development of pre-eclampsia in a pregnant woman is a medial emergency as it can result in catastrophic complications to the mother and fetus. It is important to know the medications that are appropriate in pregnancy energies because cer

A middle-aged Caucasian man with hepatitis B comes to the physician because of fever, anorexia, malaise, weight loss, weakness, myalgia, and testicular pain over the past month. His blood pressure is 170/100 mmHg. Laboratory studies show:
Hb (hemoglobin):

Pulmonary
> The hallmark of polyarteritis nodosa is sparing of
pulmonary
arteritis.

Case: A 55 yo man with history of hypertension, hypercholesterolemia, diabetes, and rheumatic heart disease is brought to the emergency department because of severe left-side retrosternal chest pain radiating to the jaw, shortness of breath, and diaphores

Left main coronary artery
> The patient symptoms and ECG findings are consistent with an acute myocardial infarction. ST-elevations in V2 through V5 implies an anterior wall MI. The artery most commonly involved in anterior wall MI is the left anterior de

The _____________ gives rise to the posterior descending artery in right-dominant circulation and supplies the inferior wall of the right ventricle. ST elevations in leads II, III, and ave would be expected in an inferior wall MI

Right main coronary artery

The __________________ supplies the lateral and posterior walls of the left ventricle. ST elevations of I, aV, and V4-V6 would be expected in a left lateral MI.

Left circumflex coronary artery

Patient with a _____________ MI frequently have St segments depressions in leads V1-V4. This occurs because the EKG leads are placed anteriorly whereas the infarction is posterior in this case, so a ST elevation MI will actually look like ST depression.

posterior

Case: A 59 yo man with diabetes comes to the physician because of numbness, weakness, and pain in the bilateral calves after walking a few blocks. The symptoms resolves after stopping for few minutes. His blood pressure is 160/90 mmHg, pulse is 83/min, an

Arteries
> The man presents with signs and symptoms of intermittent claudication, the result of peripheral vascular disease (PVD) from long-term smoking and diabetes. In addition to the symptoms of claudication, one would expect to detect diminished perip

Case: A 61 yo man comes to the ED because of SOB at rest. He ran out of his medication a week ago. He has a history of uncontrolled hypertension and CKD. his medications include metoprolol, lisinopril, and furosemide. His blood pressure is 166/98 mmHg and

Impaired LV relaxation- increased LV end-diastolic pressure-normal LV end-systolic volume
> The above clinical presentation is consistent with acute decomponstated diastolic heart failure. (LVEF on an echocardiogram a month prior had been normal). Importa

The following description has _____________ (systolic/diastolic) heart failure, which is characterized by impaired LV contraction, LV dilatation (increased end-systolic and end-diastolic volumes), and increased LV end-diastolic pressure (as blood cannot b

Systolic

Case: A 74 yo man with no past medical history is diagnosed with colon cancer and undergoes resection showing pathological stage III (6/13 lymph nodes positive) disease. He is started on adjuvant chemotherapy with 5-flurouracil (5FU) and oxaliplatin. Whil

Coronary vasospasm
> This patient is presenting with symptoms suggestive of acute myocardial infarction with ST segment elevations in anterior leads on EKG. 5-flurouracil and (its orally bio-avalible equivalent) capecitabine are both associated with
coron

Case: A 1-day-old newborn is evaluated for blue discoloration of the skin, clubbing of the fingers and toes, for feeding, and severe shortness of breath. Physical examinations shows lethargy and cyanosis. ECG shows transposition of great vessels. He is gi

Prostaglandin affects the step involving directing the deoxygenated blood from the pulmonary artery to the descending aorta.

Case: A 77-yo man with a history of diabetes, HTN, and atrial fibrillation comes to the ED because of a 2-day history of right lower extremity weakness and blurred vision. He began to feel numbers and weakness in he right thigh that spread down the right

> Left main coronary artery
This patient has EKG findings suggestive of anterolateral ischemia, possibly also involving the inter ventricular septum. Leads V2 and V3 generally reprint the distribution of the left anterior descending artery and leads V4, V

_________________ are small branches off the right coronary artery. Ischemic disease in the distribution of these arteries could be presented with ST-T wave changes in leads II, III and aVF

Marginal branch artery

The _______________, often a branch of the right coronary supplies the posterior aspect of the heart. Ischemic changes are often depicted as tall R waves in leads V1-V3 and often along with ischemic changes in II, III, and aVF.

posterior descending artery

The ______________________ supplies blood to the right heart and ischemia would present with changes in leads II, III and aVF.

right coronary artery

Case: A 65 yo man is brought to the ED because of acute-onset severe aching in the chest. He is a chronic smoker. Initial ECG shows 2-mm horizontal ST segment elevation in the anterior and lateral leads. Shortly after, the patent goes into cardiac arrest

Ventricular arrhythmia
> In a patient presenting with acute myocardial infarction,
ventricular arrhythmias
are the most important cause of death within the first 48 hours, and require prompt treatment with electrical and/or pharmacologic therapy.

Case: A previously healthy 30 yo man collapse while playing basketball. Despite resuscitation efforts from bystanders, he expires. Autopsy shows cardiac enlargement with severe thickening of the interventricular septum. The LV posterior wall is normal in

beta-myosin heavy chain
> The patient described in the vignette most likely suffered from hypertrophic cardiomyopathy (HCM), a myocardial disorder leading to impaired diastolic ventricular filling and associated left ventricular outflow tract obstruction.

__________________ is a tumor suppressor whose dysfunction is associated with
endometiral carcinoma

PTEN

_____________________ gene mutations are associated with Duchenne muscular dystrophy

Dystrophin

_______________ is an oncogene associated with cervical cancer development

p53

______________ gene mutations are associated with follicle thyroid carcinomas

RAS

Case: A 50 yo man is brought to the ED after a car accident. He has no signs of external bleeding, but his blood pressure is 90/40 mmHg and his heart rate is 120 beats per minutes. Physical examination shows distended neck veins and distant heart sounds.

Pericardial effusion
> This patient most likely suffered trauma to the chest in a car accident and is presenting to the ER with hypotension and tachycardia. Furthermore, he has Beck's triad (hypotension, distended neck veins, and distant heart sounds), wh

____________________ refers to blood in the pleural space.

Hemothorax

Case: a 40 yo Caucasian man comes to the physician because of recurrent epistaxis, facial pain, cough, fever, and weight loss, anorexia, hematuria, and arthralgia over the past 3 months. Physical examination shows a saddle-nose deformity and purpuric rash

Granulomatosis with polyangiitis (GPA)
> Involvement of the upper respiratory tract, kidney and lung, along with constitutional symptoms are very suggestive of granulomatosis with polyangiitis (firmly known as Wegener's granulomatosis); the presence of po

_________________ demonstrates a bimodal distribution: young men in their late 20s and in men and women in their 60-70s. Among younger patents it is usually acute, with hemoptysis, a sudden fall in hemoglobin, fever, dyspnea, and hematuria. Hemoptysis is

Good pasture syndrome
> Unlike with Wegener's, however, Goodpasture does not affect the upper respiratory tract and is not c-ANCA positive.
> Goodpasture syndrome is an autoimmune disorder. It occurs when the immune system mistakenly attacks and destroys

__________________ may present with hematuria and arthralgia

IgA nephropathy

_____________________ presents with fever, mummer and post blood cultures

Subacute bacterial endocarditis

Case: A 35 yo woman comes to the physican because of newly diagnosed hypertension. She has bilateral knee pain, low-grade fever, and occasional lightheadedness over the past several months. She also has occasional left arm claudication when kneading bread

Takayasu's arteritis
> Takayasu's arteritis or 'pulseless disease' involves the medium/large arteries and typically affect females between 10 and 40 years old. An acute phase with fever, weight loss and change in appetite may occur.
> Portions or branches

Case: A 75 yo woman is brought to the ED because of acute anterior ST-elevation MI. Transfer to PCI capable facility is expected to take at least 6 hours. Meanwhile, she is administered thrombolytics. She develops severe refractory hypotension and PEA arr

Left anterior descending artery
> Mechanical complications are a rare but life-threatening presentation of acute STMI. They result from thrombotic occlusion of a coronary artery with ischemic necrosis and attendant weakening of its supplied structures. Me

Case: A 50 yo woman is brought to eh ED unconscious following a motor vehicle accident. She is placed on telemetry to monitor her vitals continuously, and she remains hypotensive despite receiving a bolus of fluids and being placed on IV fluids. She regai

Glossopharyngeal nerve
> This patient has a paroxysmal supra ventricular (narrow QRS) tachyarrhythmia, which resolved with carotid sinus massage. A tachycardia with regularly spaced, narrow QRS complexes with a small downward deflection immediately follow

The _________________ nerve is responsible for motor innervation of the tongue.

Hypoglossal

The ____________ nerve makes the EFFERENT limb of the carotid reflex through its cardiac branch. The afferent limb of the carotid reflex is through he glossopharyngeal nerve

Vagus

Case: A 72 yo woman is brought to the ED because of acute-onset epigastric discomfort over the past hour. She ha dyspnea. Physical examination show profuse sweating and tachycardia with a pulse of 115/min and blood pressure of 170/100 mmHg. An EKG shows 2

Completely occlusive thrombus
> This woman is experiencing an acute myocardial infarction. In women, the incidence of heart attack rises sharply after the age of 50 years. Females, diabetics and elderly patients are more likely than man to present with at

A partially occlusive thrombus presents in a manner similar to that of a completely occlusive thrombus, but would produce a ___________________________

Non ST-segment elevation MI (NSTEMI).
> In NSTEMI, ECG changes (such as ST segment depression or T-wave inversion) may or may not be seen.

_____________ usually last less than 15 minutes in duration and presents as intermittent chest pain that can occur at rest.

Prinzmetal's angina

Case: A 65 yo man is brought to the ED 4 hours after experiencing crushing substernal chest pain, shortness of breath, and sweating. EKG shows ST elevation in leads II, III and aVF. After stabilization, an anti arrhythmic agent what affects the ventricula

Phenytoin
> The patient's presentation is consistent with acute ST-elevation myocardial infarction (MI). ST-elevation MI occurs when there is reversible, transmural ischemic damage to the myocardium.
> Class IB antiarrhytmic agents including lidocaine, to

______________ is a class IC anti arrhythmic that significantly slows phase 0 depolarization with little effect on repolarization

Flecainide

________________ is a class III anti arrhythmic (K+ channel blocker) with class II (Beta-blocker) properties.
It prolongs phase-3 repolarization and thus prolongs the QT interval

Sotalol

_____________ is a nondihydropyridine Ca2+ channel blocker that slows atrioventricular nodal conduction. It has little effect on ventricular tissue.

Verapamil

Case: A 37 yo woman comes to the physician because of lesions around her eyes. These lesions have been longstanding and occur bilaterally. An image of the lesions is shown as a yellow plaques that occur near the inner cants of the eyelid, more often on th

Increased cholesterol.
> The patient has xanthelasma. Xanthelasma palpebrarum is the most common cutaneous xanthoma. They represent an accumulation of lipid-containing macrophages in the dermis. In about 50% of patients, lipid levels are increased. Howeve

Biliary outflow obstruction and rare genetic disorders (____________, ________________) result in elevated direct bilirubin whereas hemolysis or genetic disorders ( ___________________ and _________________) can cause high indirect bilirubin.
High levels

Rotor's disease, Dubin Johnson syndrome
Crigler-Najjar, Gilbert's syndrome

_______________ are white ,yellowish chalky nodules under the skin, commonly on the elbows hands and feet. They occur in people with longstanding gout, who would normally have a raised serum uric acid.

Gouty tophi

Patients with __________________ typically have, fatigue, cold intolerance, constipation, brittle hair, weight gain, and decreased relaxation of deep tendon reflexes.

Hypothyroidism

Case: A 21 yo woman, who recently migrated from China, comes to the physician complaining of extertional dyspnea, palpitations, chest pain and hemoptysis for the past 5-months. She recalls having a severe throat infection 10 years ago but was never treate

Mitral stenosis
> The patient has rheumatic heart diseases, a complication of streptococcal pharyngitis due group A beta hemolytic streptococcus infection. Rheumatic heart disease leads to chronic valvular abnormalities with mitral stenosis being the most

Case: A 15 yo boy with history of streptococcal infection 3 weeks prior is brought to the physician by his father because of recurring chest pain, excessive fatigue, heart palatines, SOB and a thumping sensation in the chest. Physical examination shows ta

A holosystolic mummer in the let 5th intercostal space mivd-claviular line
> The patient in the vignette has rheumatic heart disease. The valve most commonly affected is the mitral valve, located in the
left 5th intercostal space, mid-clavicular line
. Mi

Case: A 24 yo African-American man collapses and dies while playing basketball. His father had also died prematurely in his thirties. A comprehensive autopsy is performed. Which is most likely diagnosis?

Hypertrophic cardiomyopathy
> Hypertrophic cardiomyopathy (or as it also known, hypertrophic obstructive cardiomyopathy, or asymmetric septal hypertrophy or idiopathic hypertrophic sub aortic stenosis) is inherited in an autosomal dominant manner in great

__________________ is the most commonly found cardiomyopathy. It results in progressive heart failure with dilations of the cardiac chambers

Dilated cardiomyopathy

____________________ is an uncommon form of cardiomyopathy, which is caused by various conditions that result in impaired cardiac filling during diastole.

Restrictive cardiomyopathy

Case: A 62 yo diabetic woman comes to the ED because of generalized malaise and epigastric pain over the past 2 hours. She is hypotensive with a blood pressure of 90/55 mmHg and pulse of 110/min. An ECG shows 2-mm convex upward ST-segment elevation in lea

Left circumflex artery
> This patient is presenting with acute ST elevation myocardial infarction (STEMI). Patients typically have predominant ST-segment elevation in leads I, all, V5 and V6 is consistent with a lateral MI, usually due to involvement of t

The ___________________________ supplies the anterior portions of the right and left ventricles as well as the anterior two-thirds of the inter ventricular septum. Occlusions of this artery would show ST elevations in V1-V2

Anterior interventrciular artery (left anterior descending)

Patients with _________________________ occlusion typically have widespread ST elevation in leads V1-V6 and leads I and all on EKG.

Left main artery occlusion

____________________ thrombosis presents as inferior myocardial infarction, manifesting as ST segment elevation in the inferior leads: II, III, and AVF.

Right marginal artery
> Right-sided leads should also be obtained in patients with inferior MI to exclude RV infarction

Patients with _______________________ occlusion have ST elevations in V3-V4.

Distal left main artery

Case: AN otherwise healthy 35- yo man is admitted to the hostpial for testing as part of a clinical research study. Inhaled/exhaled breath analysis reveals an oxygen consumption of 350 mL/min. Pulmonary artery oxygen content is 17 mL oxygen/100 mL and the

16 mmHg min/L
Mean Arterial Pressure (MAP) = (CO*systematic vascular resistance (SVR)) + central venous pressure (CVP)
In most situations CVP can be approximated to be 0. Therefore, SVR = MAP/CO
MAP = DBP + 1/3 of pulse pressure
MAP = DBP + ((SBP - DBP)/3

UWorld: A 56 yo woman is evaluated for fatigue and dyspnea on exertion. Past medical history includes HTN and systematic sclerosis. Blood pressure is 135/80 mmHg and pulse is 68/min. Cardiac examination reveals loud second heart sounds with no murmurs. Lu

Left atrial pressure
> Pulmonary artery catheters (PACs, also called Swan-Ganz or right heart catheters) are used to diagnose pulmonary hypertension and occasionally for management of critically ill patients
> During pulmonary artery catheterization, the

uWorld: A 63 yo man comes to the ED due to dyspnea. Over the past several days, the patient has experienced progressively worsening SOB while walking his dog around the block. In addition, he could not breathe while lying in bed last night and fell asleep

Increased sympathetic nervous system activity
> In patients with heart failure, compensatory activation of the renin-angiotensin-aldesterone pathway and sympathy nervous system results in increased after load (from excessive vasoconstriction), excess flui

uWorld: A 23 yo man is brough tot he ED with a chest wound. She was in her house during a severe thunderstorm when a large tree branch feel through a window. The window was shattered and large fragments of wood and glass struck the patient. She did not lo

Right ventricle
> The right ventricle compasses most of the heart's anterior surface. A penetrating injury at the left sternal border in the fourth intercostal space would puncture the right ventricle.

uWorld: A 51 yo man is brought to the ED due to chest tightness that started 30 minutes prior to arrival. His chest discomfort is associated with SOB and nausea. The patent was shoveling snow off his driveway when his symptoms began. He has a history of H

Left circumflex artery
> Leads I and aVL corresponds to the lateral limb leads on ECG. Therefore, ST elevation or Q waves in these leads are indicative of infraction involving the lateral aspect of the left ventricle, which is supplied by the left circumf

uWORLD: A 68 yo man with a history of permanent atrial fibrillation comes to the office for follow-up. He has been having symptoms due to ineffective ventricular rate control despite aggressive medical therapy. The patient's heart rate is 125/,in and irre

>Interatrial septum near the opening of the coronary sinus
The AV node is located on the endocardial surface of the right atrium, near the insertion of the septal leaflet of the tricuspid valve and the orifice of the coronary sinus.
> Radiofrequency ablat

uWorld: A 23 yo woman is brought to the ED uncurious after a Motor vehicle accident. The patient was texting on her cell phone and was not wearing a seatbelt during the incident. She has no known medical problem and takes no medications. Her temperature i

Maxillary
> The middle meningeal artery is a branch of the *maxillary artery (one of the terminal branches of the external carotid artery), which enters the skull at the foramen spinosum and supplies the dura mater and periosteum.
> This patient has suffe

uWorld: A 45 yo man presents to the ED with a two-day history of dyspnea, orthopnea, and ankle swelling. This morning, he developed nausea , vomiting, and headache. Physical examinations reveal pulmonary crackles and bilateral pitting edema up to this kne

Increase Pressure, Increase Volume
> Ntrioprusside is a short-acting balanced venous and arterial vasodilator that decreases both preload and after load. Since these changes are balanced, stroke volume is maintained.
> This patient most likely has hyperte

uWorld: A 65 yo man reports multiple episodes of lightheadedness while buttoning a tight shirt collar. During 2 episodes, he passed out briefly but sustained no injuries. His blood pressure was 70/40 mmHg and pulse was 45/min during one of the episodes. P

Glossopharyngeal
> The patient's history is suggestive of
carotide sinus hypersensitivity
, triggered by pressure on the carotid sinus by a tight shirt collar. The carotid sinus baroreceptors are important in blood pressure control and use arterial wall s

A 78 yo woman is hospitalized due to acute MI. The patient has been having intermittent chest pain for 3 days and came to the hospital when the pain became unremitting. She has type 2 DM, but her medical follow up has been poor. On the third day of hospit

Posterior descending
>Papillary muscle rupture is a life-threatening complication that typically occurs 3-5 days after MI and presents with acute mitral regurgitation and pulmonary edema. The posteromedial papillary muscle is supplied sole by the posterio

uWORLD: A 23 yo previously health man is brought to the ED after a stab injury. His friends report that they were "Talking down the street minding our own business when a guy jumped out in front of us from behind a dumpster and stabbed him in the chest" T

left lung
> The left ventricle forms that apex of the heart and can reach as far as the fifth intercostal space at the left midclavicular line (MCL). All other chambers of the heart lie medial to the left MCL. The lungs overlap much of the anterior surfac

uWORLD: A 10 yo boy is brought to the clinic by his parents due to chronic fatigue, SOB and failure to gain weight. The child was recently adopted from an orphanage in Asia. He weights 20 kg (44.2 lb). Vitals signs are within normal limits. Physical exami

Increasing systemic vascular resistance
> In patients with Tetralogy of Fallot, squatting during a Tet spell increases systemic vascular resistance and decreases right-toleft shunting, thereby increasing pulmonary blood flow and improving oxygenation stat

uWorld: A 67 yo man is brought to the ED by his son after a syncopal episode. The son was helping his father clean out his garage when his father complained of dizziness. As his son was helping him into a chair, the patient lost consciousness. He woke up

AV node
> Cells throughout the conduction system posses pacemaker ability. The SA node, however, normally stiles impulse generation by other cells in the conduction system as it is able to fire more rapidly. Should impulse conduction form the SA node be i

uWorld: A 43 yo man is evaluated for occasional palpations provoked by anxiety. The patent describes a sudden-onset pounding sensation int he chest followed by lightheadedness and SOB. He has never had syncope. The patent has no history of heart disease,

Slowed spontaneous depolarization
> Class IV antiarrhytmatics (eg. verapamil, diltiazem) are commonly used to prevent recurrent nodal arrhythmias (eg. paroxysmal supraventciular tachycardia). They work by blocking calcium channels in slow-response cardiac

uWorld: A 44 yo man with progressive dyspnea is diagnosed with dilated cardiomyopathy. Despite optimal medical therapy, he continues to have symptoms and diseases progression is noted. He undergoes cardiac transplantation after a suitable donor becomes av

Na+/Ca2+ exchanger
> Calcium efflux from cardiac cells prior to relaxation is primarily mediated via an Na+/Ca2+ exchange pump and sarcoplasmic reticulum Ca2+-ATPase pump

uWORLD: A 47 yo man is brought to the ED after being involved in a high-speed motor vehicle collision. He was a restrained driver and rear-ended a slow-moving car on the highway. He complains of chest pain, abdominal pain, and difficulty breathing. The pa

Aortic isthmus (right after arch of aorta and beforedesneding aorta)
Traumatic aortic rupture is most often caused by the rapid deceleration that occurs in motor vehicle collisions. The most common site of injury is the aortic isthmus, which is tethered b

uWORLD: A 32 yo man comes to the office due to progressive dyspnea, dizziness, and chest discomfort. He has no prior medical problems but had a mild respiratory illness 2 weeks earlier that resolved spontaneously. His father has a history of myocardial in

Drop in pulse amplitude during inspiration.
> This patient has a pericardial effusion with associated
Cardiac tamponade
. Classic signs of cardiac tamponade include, HTN, elevated jugular venous pressure, and muffled heart sounds (Beck triad). ECG will sh

uWorld: A 38 yo woman is being evaluated for SOB. The patient has had poor exercise tolerance for the last several months that she initially attributed to reconditioning. Over the last several weeks, her symptoms have gotten progressively worse and she cu

Mitral stenosis
> Under normal circumstances, pulmonary capillary wedge pressure (PCWP) closely reflect left atrial (LA) and left ventricular end-diastolic pressure (LVEDP). Mitral stenosis leads to an increase in the LA pressure that is reflected as elev

uWORLD: A 45 yo man comes to the ED with fever, SOB, and generalized weakness. Two days ago, the patient was diagnosed with a urinary tract infection and treated with oral antibiotics. He has a history of coronary artery disease and underwent coronary art

Normal saline infusion
> Pressure-volume loops represent the relationship between pressure and volume in the left ventricle during systole and diastole. An increase in the circulating volume increases preload (left ventricular end-diastolic volume) and ca

uWORLD: A 63 yo man with long-standing hypertension has undergone multiple changes in his antihypertensive regimen over the years with only minimal improvement in his blood pressure. A CT study of his thoracolumbar spine is performed for evaluation of chr

50%
> Blood flow is directly proportional to the vessel radius raised to the fourth power. Resistance to blood flow is inversely proportional to the vessel radius raised to the fourth power.

uWORLD: A 43 yo man comes to the ED after experiencing 4 episodes of coffee ground emesis that started earlier this morning. He also describes epigastric pain over the last 3-4 months that was relieved by over-ta-counter antacids. He as no other past medi

End-diastolic sarcomere length
> Intravenous fluids increase the intravascular and left ventricular end-diatostyic volumes. The increase in preload stretches the myocardium and increases the end-diastolic sarcomere length, leading to an increase in stroke

uWORLD: A 53 yo man comes to the office due to diffulity breathing and increasing fatigue. He has been sleeping in a recliner to receive his SOB at night. Past medical history is significant for HTN and hyperlipidemia. Two months ago, he suffered a MI tha

Listening at end expiration
> The third heart sound (S3) occurs during early diastole and is best heard with the bell of the stethoscope over the cardiac apex while the patient is in the left lateral decubitus position at end expiration. It can be a norma

uWorld: A 46 yo man comes to the office with chest pain and dyspnea on exertion. he has no known medical problems and leads a sedentary lifestyle. He is a lifetime nonsmoker. Noninvasive cardiac testing is non diagnostic. Left and right heart catheterizat

Right ventricle
> Right-sided pressures in the heart are lower than left-sided pressures due to lower resistance in the pulmonary vasculature. Right ventricular diastolic pressure is similar to right atrial/central venous pressure (1-6 mmHg), whereas pulm

uWorld: A 55 yo man comes to the office for following-up HTN and Contrary heart disease. He has been on several medications in the past with suboptimal blood pressure control. The physician considers adding verapamil to the patient's current regimen, as i

Little dependence on extracellular calcium
> Contraction initiation in cardiac and smooth muscle cells is dependent on extracellular calcium influx through L-type calcium channels, which can be prevented by calcium channel blockers (eg, verapamil). Skelet

uWORLD: A 32 yo man is evaluated in the ED due to fever, night sweats and chills over the last several days. The patient has been using intravenous drugs recently as he is "stressed out". He has otherwise been in good health with no medical problems. Temp

Flow from the aortic root to the right ventricle continuously
>During the normal cardiac cycle, central aortic pressure is higher than right ventricular pressure during systole and diastole. Consequently, an intracardiac fistula between the aortic root an

uWORLD: A 23 yo man comes to the ED with sudden onset of heart palpitations that started while he was a his desk at work. The patient has no known medical problems and does not use tobacco or illicit drugs. He drinks alcohol occasionally on the weekends.

Prolonged atrioventricular node refractory period
> Carotid sinus massage leads to an increase in parasympathetic tone causing temporary inhibition of SA node activity, slowing of condition through the AV node, and prolongation of the AV node refractory p

uWORLD: A 32 yo woman is recruited for a research study. She is evaluated in the hospital as part of the study protocol. Her medical history includes an uncomplicated appendectomy, and medications include oral contraceptive pills and a multivitamin. Durin

Exercise
> The cardiorespiratory response to exercise includes increased HR, CO and RR in order to balance the increased total tissue oxygen consumption and CO2 production. ^These coordinated adaptations result in relatively constant arterial blood gas va

uWORLD: A 54 yo man is being evaluated for exertion dyspnea that started 6 months ago. The patient has no associated cough or wheezing. There is no known history of heart disease in his family. ECG, chest x-ray, exercise stress test, and ECG are obtained

Rate of oxygen consumption
> Cardiac output = Stroke volume X Hear rate
> Fick principle: Cardiac output = rate of O2 consumption/ (arteriovenous O2 content difference)

uWORLD: Physiologist conducting research on the electrical properties of the heart measure AP conduction velocity at 4 different points within normal cardiac tissue. The results, expressed in terms of speed conduction are the following
Point 1: 0.05 m/sec

AV node, ventricular muscle, atrial muscle, Purkinje system
> Thecardiac action potential conduction speed is slowest in the AV nod and fastest in the Purkinje system. Conduction speed of the atrial muscle is faster than that of the ventricular muscle.

uWORLD: A 66 yo man with non-ischemic cardiomyopathy has persistent SOB despite being on maximal heart failure therapy. He has no lightheadedness or chest pain. Blood pressure is 133/72 mmHg and pulse is 76/min. The patient is treated with a novel medicat

natriuretic peptides
> Endogenous natriuretic peptides promote vasodilation and natriuresis and provide a beneficial counter-regulatory mechanisms in patients with heart failure. They are metabolized by the enzyme neprilysin, and inhibition of nephrilysin

uWORLD: A 34 yo woman who recently emigrated from Russia comes to the physician complaining of weakness, exertion dyspnea, and orthopnea. On cardiac auscultation, a snap followed by a rumbling diastolic mummer is heard over the cardiac apex. The snap most

D - Mitral valve opening
> The patient most likely has mitral stenosis, which often results in an extra heart sound (termed an opening snap) being heard shortly after the S2 heart sound. (point C). The opening snap results forms he abrupt halting of leafl

uWORLD: A 24 yo primigravida women at 36 wee gestation comes to the physician complaining of light headedness and nausea at bedtime. Her pregnancy has been uncomplicated and her past medical history is unremarkable. She takes one multivitamin and one iron

Decreased venous return
> Supine hypertension syndrome (or aortocaval compression syndrome) is characterized by hypertension, pallor, sweating, nausea, and dizziness the occur when a pregnant women lies supine (on her back). Symptoms resolve with sitting,

uWORLD: A 43 yo man reports occasional chest discomfort over the last 6 weeks. He thinks it is most likely musculosketla pain but is concerned due to family history of heart disease. The patient has no medical problems and does not smoke cigarettes. He le

Duration of diastole
> There is a normal cyclical variation in the coronary blood flow pattern during a cardiac cycle, with the maximum flow occurring during diastole and decreased blood flow with ventricular systole. Shorter duration of diastole during t

uWORLD: A 43 yo man is being evaluated for occasional retrosternal chest pressure that develops with moderate exertion and sometimes occurs when resting. He does not use alcohol, tobacco, or illicit drugs. The patient has an extensive family history of co

Arginine
> Nitric oxide is synthesized from arginine by nitric oxide synthase. As a precursor of nitric oxide, arginine supplementation may play an adjunct role in the treatment of conditions that improve with vasodilation, such as stable angina.

uWORLD: A 46 yo man comes to the office for an annual examination. He has a uncomfortable heartbeat sensation at night that he tries to decrease by sleeping on his right side. The patient has bad mild shortness of breath with exertion over the last 6 mont

Aortic regurgitation
> This presentation is most suggestive of
aortic regurgitation
. The inability of aortic valve leaflets to effectively close during diastole leads to regurgitation of blood back into the left ventricular (LV) cavity with an increase i

uWORLD: A 48 y/o man with diet-controlled type 2 diabetes mellitus is being evaluated for occasional retrosternal chest pain. He has no history of HTN, and his blood pressure measurements during office visits were always within normal limits. There is no

Total systemic vascular resistance
> Left ventricular (LV) end-diastolic volume and pressure both rise during exercise due to an increase in LV filling (increased venous return) by peripheral vasoconstriction and the pumping actions of actively contractin

An increase in effective stroke volume or ejection fraction is depicted on the left ventricular pressure-volume relationship by _________________ of the loop with a shift in the isovolumic relaxation line to the left (indicating less residual blood volume

widening

Acetylcholine and adenosine ___________ the rate of spontaneous depolarization in the cardiac pacemaker cells by ___________ phase 4

reduce; prolonging

A 78 yo man is evaluated for periodic headaches relieved by over-the-counter acetaminophen. He takes no other medications and is a lifelong nonsmoker. The patient's blood pressure is 180/70 mmHg, and pulse is 75/min and regular. During a prior office visi

Aortic stiffening
> The patient's BP reading shows a pattern of elevated Systolic blood pressure (SBP) with normal (<90 mmHg) diastolic BP, constant with
isolated systolic hypertension (ISH)
. ISH is often seen in patients age >60, and is responsible for

uWORLD: A 44 yo man with non-ischemic cardiomyopathy experiences fatigue and exertion dyspnea. He has no symptoms at rest, but his daily activities are significantly limited by dyspnea. He is compliant with dietary and exercise recommendations and takes h

Funny sodium channels during phase 4
> Ivavradine selectively inhibits the funny sodium channel (If) prolonging the slow depolarization phase (phase 4) and slowing the sinoatrial node firing rate. It has a negative chronotropic effect with no effect on ca

A 56 yo man comes to the ED due to chest palpitations. He feels that his heartbeat is fast and very irregular. The patient has no chest pain, SOB, or dizziness. Physical examination confirms that presence of an irregularly regular rhythm with a heart rate

Absent P waves
> The patient's presentation (palpitations, tachycardia, and irregularly irregular rhythm) is suggestive of
atrial fibrillation (AF).
The ECG in patients with AF typically shows an
absence of P waves
and
irregularly irregular
rhythm with
va

uWorld: A 69 yo man is brought to the ED with sudden onset palpation and dyspnea. His past medical history is significant for hypertension and GI reflux disease. An ECG reveals a heart rate of 120 ppm with an irregular rhythm, narrow QRS complexes, and no

AV node refractory period
> This patients ECG findings are diagnostic for
atrial fibrillation (AF) with rapid ventricular response.
Ventricular response
in AF is dependent on the transmission of abnormal atrial impulses through the
AV node.*. Each time th

uWORLD: In an experiment, 3 L of isotonic saline are infused IV into a healthy volunteer after multiple baseline physiologic parameters are recored. Serial blood pressure measurements show an increase in the systolic and diastolic blood pressure. Ultrason

Glomerular filtration rate
> Atrial natriuretic peptide and brain natriuretic peptide are pleased from the atria and ventricles, respectively, in response to myocardial wall stretch due to intravascular volume expansion. These endogenous hormones promote

uWolrd: A 44 yo man with nonischemic cardiomyopathy and chronic systolic heart failure comes to the cardiology clinic for a follow-up appointment. The patient has experienced recent worsening of dyspnea on exertion, and now has to stop halfway to catch hi

Atrial cardiomyocytes
> ANP is screwed by atrial cardiomyocytes in response to atrial stretch induced by hypertension or hypervolemia. ANP causes peripheral vasodilation and increased urinary excretion of sodium and water. Neprilysin inhibitors (eg, sacub

uWORLD: A multinational research institute conducting experiments on human circulatory physiology enrolls a healthy 30- yo male volunteer to assess the oxygen consumption rates of various organs. During the study, the blood oxygen content of the aorta and

Coronary sinus
> Myocardial oxygen extraction exceeds that of any other tissue or organ in the body. Due to the high degree of oxygen extortion, increases in myocardial oxygen demand (eg, during exercise) are met by a nearly proportionate increase in coro

A 44 yo man is hospitalized with multiple injuries following a motor vehicle accident. An arteriovenous shunt created by the injury would most likely result in which of the following changes in the left ventricular pressure-volume loop?

Decrease is after load, increase in preload
> Arteriovenous (AV) shunts can be congenital or acquired; acquired forms can result from medical intervention or penetrating injuries. AV shunts increase preload and decrease afterload by routing blood directly

uWorld: A 43 yo lomean comes to the ED due to nausea and vomiting with left-sided chest pain. She states that the pain began 30 minutes ago while playing with her children. She has a history of dyslipidemia and hypertension. The patient's mother died from

Blood flow per minute
>The circulatory system is a continuous circuit, and therefore the volume output of the left ventricle must closely match the output of the right ventricle. This balance is necessary to maintain continuous blood flow through the body

Incomplete fusion of atrial septum primum and second secundum causes _________________
Aplasia of the atrial septum secundum causes _________________

ASD ; Foramen ovale
> Although AsD may lead to paradoxical embolism, it is much less common than PFO and characterized by
fixed splitting of S2
. Unprepared ASDs may lead to heart failure, pulmonary hypertension, or Eisenmenger syndrome

This young patient has a
cryptogenic stroke
, which is evaluated with echocardiography with a "bubble study" to identify right-to-left intracardiac shunts.
This study is performed by injecting agitated normal saline intravenously and observing for micro b

patent foramen ovale (PFO) ; atrial septal defect (ASD)

Subclavian steal syndrome occurs due to stenosis of the proximal ______________, which leads to reversal in blood flow from the
contralateral vertebral artery
to the
ipsilateral vertebral artery
.
Patients may have symptoms related to arm ischemia in the

subclavian artery

This patient's presentation is consistent with ___________________, which typically occurs due to hemodynamically significant stenosis of the subclavian artery proximal to the origin of the vertebral artery.
The lowered distal subclavian arterial pressure

Subclavian steal syndrome

The signs and symptoms of Superior Vena Cava (SVC) syndrome are similar to those of obstructed ________________, except that both sides of the faces, neck, and chest and both arms would be involved.

right brachiocephalic (innominate) vein
> Brachiocephalic vein obstruction cause symptoms similar to those seen in SVC syndrome, but only on one side of the body.

Streptococcus gallolyticus (formerly S bovis), endocarditis and bacteremia are associated with _________________ in 25% of cases.

gastrointestinal lesions (colon cancer)
> When S. gallolyticus is cultured in the blood, workup for colonic malignancy with colonoscopy is essential.
> It causes a subacute bacterial endocarditis with symptoms to those of S. viridian's SBE.

The ____________ is derived from the 3rd aortic arch.

Common carotid artery

The ___________ is derived from the 4th aortic arch

Subclavian arteries

Vascular access during
cardiac catheterization
typically obtained through either the
common femoral artery
or radial artery.
The common femoral artery is the continuation of the external iliac artery as it crosses the inguinal ligament.
Arterial
puncture

Retroperitoneal hemorrhage
> Bleeding in the retroperitoneal space
Cannot be controlled
with external compression and can lead to life-threatening hemorrhage. Affected patients typically develop
hemodynamic instability
with significant hypotension, a drop

The SA node is a compact subepicardial structure that consist of specialized pacemaker cells located at the _______________

junction of the right atrium and superior vena cava

Atrial fibrillation is associated with increased risk of systemic thromboembolism. The __________ is the most common site of thrombus formation.

Left atrial appendage
> The left atrial appendage (LAA) is a small saclike structure in the left atrium that is particularly susceptible to thrombus formation. Approximately 90% of left atrial thrombi are found within the LAA in patients with nonvalvular

Tetralogy of Fallot results form anterior and cephalic deviation of the __________________ during embryologic development, resulting in a maligned ventricular septal defect (VSD) with an overriding aorta,
As a result, the patient has right ventricular out

infundibular septum

This patient with
deep venous thrombosis (DVT) complicated by
pulmonary embolism* has had recurrent episodes of Gi hemorrhage, including a recent one requiring prolonged hospitalization. Given the risk of bleeding associated with anticoagulation in this p

Inferior vena cava (IVC) filter
> IVC filters are designed to prevent the propagation of DVT form the legs tot he lung vasculature and are used in patients who have
contraindicaitons to anticoagulation

Pressure in the left renal vein may become elevated due to compression where the vein crosses the aorta beneath the superior mesenteric artery. This "nutcracker effect" can cause hematuria and flank pain.
Pressure can also be elevated in the left gonadal

varicocele

On posteroanterior chest x-ray, the right middle lobe is seen adjacent to the right border of the heart, which is primarily formed by the right atrium.
Consolidation in the right middle lobe can obscure the x-ray silhouette of the _____________

right heart border

The femoral triangle (lateral to medial) consists of the _________, ________, ________ and __________. Cannulation of the femoral vein should occur approximately 1 cm below the inguinal ligament and just medial to the femoral artery pulsation.

> femoral nerve
> femoral artery
> femoral vein
> deep inguinal node/lymphatic vessels

The esophagus lies within closest proximity of what part of th heart?

Left atrium
> Due to its adjacent proximity, conditions that result in left atrial enlargement (eg. atrial fibrillation, mitral stenosis) can cause dysphagia through external compression of the esophagus
> The left atrium forms the majority of the posteri

The ___________ lies posterior to the esophagus and the left atrium.

descending thoracic aorta
> Transesophageal echocardiography, allows for the detection of abnormalities such as dissection or aneurysm.

The inferior wall of the left ventricle forms most of the inferior (diaphragmatic) surface of the heart and is supplied by the posterior descending artery. In 85%-90%, the posterior descending artery derives from the _______________(right dominant coronar

right coronary artery

The intimal tear in Stanford type A aortic dissection (involving the _____________) suavely originates in the sinotubular junction
whereas the intimal flap in Stanford type B aortic dissection usually starts near the origin of the __________________.
Diss

> ascending aorta
> left subclavian artery

The great saphenous vein is the located superficially in the leg and is the longest vein in the body.
Surgeons access the great saphenous vein in the medial leg or, less commonly, near its point of termination in the _____________ of the upper thigh.

femoral triangle
> The femoral triangle is bordered by the inguinal ligament superiorly, sartorial muscle laterally, and adductor longus muscle medially.
> Its drains into the femoral vein within the region of the femoral triangle, a few centimeters infer